Saltar al contenido principal
LibreTexts Español

3.1: Definición de la Integral

  • Page ID
    110482
  • \( \newcommand{\vecs}[1]{\overset { \scriptstyle \rightharpoonup} {\mathbf{#1}} } \) \( \newcommand{\vecd}[1]{\overset{-\!-\!\rightharpoonup}{\vphantom{a}\smash {#1}}} \)\(\newcommand{\id}{\mathrm{id}}\) \( \newcommand{\Span}{\mathrm{span}}\) \( \newcommand{\kernel}{\mathrm{null}\,}\) \( \newcommand{\range}{\mathrm{range}\,}\) \( \newcommand{\RealPart}{\mathrm{Re}}\) \( \newcommand{\ImaginaryPart}{\mathrm{Im}}\) \( \newcommand{\Argument}{\mathrm{Arg}}\) \( \newcommand{\norm}[1]{\| #1 \|}\) \( \newcommand{\inner}[2]{\langle #1, #2 \rangle}\) \( \newcommand{\Span}{\mathrm{span}}\) \(\newcommand{\id}{\mathrm{id}}\) \( \newcommand{\Span}{\mathrm{span}}\) \( \newcommand{\kernel}{\mathrm{null}\,}\) \( \newcommand{\range}{\mathrm{range}\,}\) \( \newcommand{\RealPart}{\mathrm{Re}}\) \( \newcommand{\ImaginaryPart}{\mathrm{Im}}\) \( \newcommand{\Argument}{\mathrm{Arg}}\) \( \newcommand{\norm}[1]{\| #1 \|}\) \( \newcommand{\inner}[2]{\langle #1, #2 \rangle}\) \( \newcommand{\Span}{\mathrm{span}}\)\(\newcommand{\AA}{\unicode[.8,0]{x212B}}\)

    La integral que estudiaste en cálculo es la {}, llamada así por el matemático alemán, quien proporcionó una formulación rigurosa de la integral para

    reemplazar la noción intuitiva debido a y. Desde la época de Riemann se han definido y estudiado otros tipos de integrales; sin embargo, todas son generalizaciones de la integral de Riemann, y apenas es posible entenderlas o apreciar las razones para desarrollarlas sin una comprensión profunda de la integral de Riemann. En esta sección tratamos funciones definidas en un intervalo finito\([a,b]\). A {} es un conjunto de subintervalos\ [\ begin {ecuación}\ label {eq:3.1.1} [x_0, x_1],\ [x_1, x_2],\ dots, [x_ {n-1}, x_n], \ end {ecuación}\] donde\ [\ begin {ecuación}\ label {eq:3.1.2} a=x_0<x_1\ cdots<x_n=b. \ end {ecuación}\] Así, cualquier conjunto de\(n+1\) puntos que satisfaga define una partición\(P\) de\([a,b]\), que denotamos por\ [ P=\ {x_0, x_1,\ dots, x_n\}. \] Los puntos\(x_0\),\(x_1\),,\(x_n\) son los {} de\(P\). La mayor de las longitudes de los subintervalos es la {} de\(P\), escrita como\(\|P\|\); así,\ [ \ |P\ |=\ max_ {1\ le i\ le n} (x_i-x_ {i-1}). \] Si\(P\) y\(P'\) son particiones de\([a,b]\), entonces\(P'\) es un {} si cada punto de partición de\(P\) es también un punto de partición de\(P'\); es decir, si\(P'\) se obtiene insertando puntos adicionales entre los de\(P\). Si\(f\) se define on\([a,b]\), entonces una suma\ [ \ sigma=\ suma_ {j=1} ^n f (c_j) (x_j-x_ {j-1}), \] donde\ [ x_ {j-1}\ le c_j\ le x_j,\ quad 1\ le j\ le n, \] es a. (Ocasionalmente vamos a decir más simplemente que\(\sigma\) es una suma de Riemann de\(f\) más\([a,b]\).) Dado que se\(c_j\) puede elegir arbitrariamente en\([x_j,x_{j-1}]\), hay infinitamente muchas sumas de Riemann para una función dada\(f\) sobre una partición dada\(P\).

    Te dejamos a ti (Ejercicio~) demostrar que\(\int_a^b f(x)\,dx\) es único, si existe; es decir, no puede haber más de un número\(L\) que satisfaga Definición~.

    Por brevedad diremos integrable” e integral''cuando nos referimos a Riemann integrable” y Riemann integral”. Decir que\(\int_a^b f(x)\,dx\) existe equivale a decir que\(f\) es integrable en\([a,b]\).

    -.3em Una aplicación importante de la integral, en efecto, la que invariablemente se utiliza para motivar su definición, es el cálculo del área delimitada por una curva\(y=f(x)\), el\(x\) eje -y las líneas\(x=a\) y\(x=b\) (``el área bajo la curva”), como en la Figura~.

    6pt

    12pt

    Por simplicidad, supongamos que\(f(x)>0\). Entonces\(f(c_j)(x_j-x_{j-1})\) está el área de un rectángulo con base\(x_j-x_{j-1}\) y altura\(f(c_j)\), por lo que la suma de Riemann \ [\ sum_ {j=1} ^n f (c_j) (x_j-x_ {j-1}) \] puede interpretarse como la suma de las áreas de rectángulos relacionados con la curva\(y=f(x)\), como se muestra en la Figura ~.

    6pt

    12pt

    Un argumento aparentemente plausible, que las sumas de Riemann se aproximan cada vez más al área bajo la curva a medida que aumenta el número de rectángulos y la mayor de sus anchuras se hace más pequeña, parece apoyar la aseveración que\(\int_a^b f(x)\,dx\) iguala al área bajo la curva. Este argumento es útil como motivación para la Definición~, que sin ella parecería misteriosa. Sin embargo, la lógica es incorrecta, ya que se basa en el supuesto de que el área bajo la curva ha sido previamente definida de alguna otra manera. Si bien esto es cierto para ciertas curvas como, por ejemplo, las que consisten en segmentos de línea o arcos circulares, no lo es en general. De hecho, el área bajo una curva más complicada es {} para ser igual a la integral, si la integral existe. Que esta nueva definición sea congruente con la antigua, donde se aplica esta última, es evidencia de que la integral proporciona una generalización útil de la definición de área.

    6pt

    12pt

    6pt

    12pt

    Vamos a mostrar que si\(f\) es unacounded on\([a,b]\),\(P\) es alguna partición de\([a,b]\), y\(M>0\), entonces hay sumas de Riemann\(\sigma\) y\(\sigma'\) de\(f\) sobre\(P\) tal que\ [\ begin {ecuación}\ label {eq:3.1.7} |\ sigma-\ sigma'|\ ge M. \ end {ecuación}\] Dejamos a usted (Ejercicio~) para completar la prueba demostrando de esto que\(f\) no puede satisfacer Definición~.

    Que\ [ \ sigma=\ sum_ {j=1} ^nf (c_j) (x_j-x_ {j-1}) \] sea una suma de Riemann de\(f\) más de una partición\(P\) de\([a,b]\). Debe haber un entero\(i\) en\(\{1,2, \dots,n\}\) tal que\ [\ begin {ecuación}\ label {eq:3.1.8} |f (c) -f (c_i) |\ ge\ frac {M} {x_i-x_ {i-1}} \ end {ecuación}\] para algunos\(c\) in\([x_{i-1}x_i]\), porque si no hubiera así, tendríamos\ [ |f (x) -f (c_j) |<\ frac {M} {x_j-x_ {j-1}},\ quad x_ {j-1}\ le x\ le x_j,\ quad 1\ le j\ le n. \] Entonces\ [\ begin {eqnarray*} |f (x) |\ ar=|f (c_j) +f (x) -f (c_j) |\ le|f (c_j) |+|f (x) -f (c_j) |\\ \ le\ ar |f (c_j) |+\ frac {M} {x_j-x_ {j-1}},\ quad x_ {j-1}\ le x\ le x_j,\ quad 1\ le j\ le n. \ end {eqnarray*}\] lo que implica que\ [ |f (x) |\ le\ max_ {1\ le j\ le n} |f (c_j) |+\ frac {M} { x_j -x_ {j-1}}, \ quad a\ le x\ le b, \] contradiciendo la suposición\(f\) que no tiene límites en\([a,b]\).

    Ahora supongamos que eso\(c\) satisface, y considera la suma de Riemann \ [\ sigma'=\ sum_ {j=1} ^nf (c'_j) (x_j-x_ {j-1}) \] sobre la misma partición\(P\), donde\ [ c'_j=\ left\ {\ casespace\ begin {array} {ll} c_j, &j\ ne i,\\ c, &j=i.\ end {array}\ right. \]

    Dado que\ [ |\ sigma-\ sigma'|=|f (c) -f (c_i) | (x_i-x_ {i-1}), \] implica.

    Debido al Teorem~, consideramos solo funciones acotadas a lo largo del resto de esta sección.

    Para demostrar directamente desde Definition~ que\(\int_a^b f(x)\,dx\) existe, es necesario descubrir su valor\(L\) de una forma u otra y demostrar que\(L\) tiene las propiedades que requiere la definición. Para una función específica puede suceder que esto se pueda hacer por cálculo sencillo, como en Ejemplos~ y. Sin embargo, esto no es así si el objetivo es encontrar condiciones generales que impliquen que eso\(\int_a^b f(x)\,dx\) existe. El siguiente enfoque evita la dificultad de tener que descubrir de\(L\) antemano, sin saber si existe en primer lugar, y sólo requiere que comparemos dos números que deben existir si\(f\) está acotado en\([a,b]\). Veremos que\(\int_a^b f(x)\,dx\) existe si y sólo si estos dos números son iguales.

    Si\(m\le f(x)\le M\) para all\(x\) in\([a,b]\), entonces\ [ m (b-a)\ le s (P)\ le S (P)\ le M (b-a) \] para cada partición\(P\); así, el conjunto de sumas superiores de\(f\) sobre todas las particiones\(P\) de\([a,b]\) está acotado, como es el conjunto de sumas inferiores. Por lo tanto, Teoremas ~\(\overline{\int_a^b}f(x)\,dx\) e implican que y\(\underline{\int_a^b}f(x)\,dx\) existen, son únicos, y satisfacen las desigualdades\ [ m (b-a)\ le\ overline {\ int_a^b} f (x)\, dx\ le M (b-a) \] y\ [ m (b-a)\ le\ subrayan {\ int_a^b} f (x)\, dx\ le M (b-a). \]

    Si\(P=\{x_0,x_1, \dots,x_n\}\), entonces\ [ S (P) =\ suma_ {j=1} ^n m_J (x_j-x_ {j-1}), \] donde\ [ m_J=\ sup_ {x_ {j-1}\ le x\ le x_j} f (x). \] Una suma arbitraria de Riemann de\(f\) over\(P\) es de la forma\ [ \ sigma=\ sum_ {j=1} ^n f (c_j) (x_j-x_ {j-1}), \] donde\(x_{j-1}\le c_j\le x_j\). Ya que\(f(c_j)\le M_j\), se deduce que\(\sigma\le S(P)\).

    Ahora vamos\(\epsilon>0\) y elegimos\(\overline c_j\)\([x_{j-1},x_j]\) para que\ [ f (\ overline c_j) > m_j -\ frac {\ epsilon} {n (x_j-x_ {j-1})},\ quad 1\ le j\ le n. \] La suma de Riemann producida de esta manera es\ [ \ overline\ sigma=\ sum_ {j=1} ^n f (\ overline c_j) (x_j-x_ {j-1}) >\ suma_ {j=1} ^n\ izquierda [M_j-\ frac {\ epsilon} { n (x_j-x_ {j-1})})\ derecha] (x_j-x_ {j-1}) =S (P) -\ épsilon. \] Ahora Teorem~ implica que\(S(P)\) es lo supremo del conjunto de Riemann sumas de\(f\) más\(P\).

    Ejercicio~.

    \ begin {ejemplo}\ rm Vamos\ [ f (x) =\ left\ {\ casespace\ begin {array} {ll} 0&\ mbox {si $x$ es irracional},\\ 1&\ mbox {si $x$ es racional},\ end {array}\ right. \] y\(P=\{x_0,x_1, \dots,x_n\}\) ser una partición de\([a,b]\). Dado que cada intervalo contiene números tanto racionales como irracionales (Teoremas ~ y),\ [ m_j=0\ mbox {\ quad y\ quad} m_J=1,\ quad 1\ le j\ le n. \] Por lo tanto,\ [\ begin {eqnarray*} S (P)\ ar=\ sum_ {j=1} ^n1\ cdot (x_j-x_ {j-1}) =b-b a\ \\ arraytext {y}\\ s (P)\ ar=\ suma_ {j=1} ^n0\ cdot (x_j-x_ {j-1}) =0. \ end {eqnarray*}\] Dado que todas las sumas superiores son iguales\(b-a\) y todas las sumas inferiores iguales\(0\), Definición~ implica que \ [\ overline {\ int_a^b} f (x)\, dx=b-a\ mbox {\ quad y\ quad} \ subrayado {\ int_a^b} f (x)\, dx=0. \]\ end {ejemplo}

    La motivación para la Definición~ se puede ver volviendo a considerar la idea de área bajo una curva. Figura~ muestra la gráfica de una función positiva\(y=f(x)\),\(a\le x\le b\), con\([a,b]\) particionada en cuatro subintervalos.

    6pt

    12pt

    Las sumas superior e inferior de\(f\) sobre esta partición pueden interpretarse como las sumas de las áreas de los rectángulos coronadas por las líneas continuas y discontinuas, respectivamente. Esto indica que una definición sensible de área\(A\) bajo la curva debe admitir las desigualdades\ [ s (P)\ le A\ le S (P) \] para cada partición\(P\) de\([a,b]\). Así,\(A\) debe ser un límite superior para todas las sumas inferiores y un límite inferior para todas las sumas superiores de\(f\) sobre particiones de\([a,b]\). Si\ [\ begin {ecuación}\ label {eq:3.1.11} \ overline {\ int_a^b} f (x)\, dx=\ subrayado {\ int_a^b} f (x)\, dx, \ end {ecuación}\]

    sólo hay un número, el valor común de las integrales superior e inferior, con esta propiedad, y definimos\(A\) que sea ese número; si no se mantiene, entonces no\(A\) se define. A continuación veremos que esta definición de área es congruente con la definición señalada anteriormente en términos de sumas de Riemann.

    El {} es una generalización importante de la integral de Riemann. Aquí la definimos, pero limitamos nuestro estudio de la misma a los ejercicios de esta y otras secciones de este capítulo.

    \ begin {exerciselist}

    Demostrar que no puede haber más de un número\(L\) que satisfaga Definición~.

    2pt

    Supongamos que eso\(\int_a^bf(x)\,dx\) existe y hay un número\(A\) tal que\(\delta>0\), por cada\(\epsilon>0\) y, hay una partición\(P\) de\([a,b]\) con\(\|P\|<\delta\) y una suma Riemann\(\sigma\) de\(f\) sobre\(P\) que satisface la desigualdad\(|\sigma-A|<\epsilon\). \(\int_a^b f(x)\,dx=A\)Demuéstralo.

    2pt

    Demostrar directamente desde Definición~ que \ [\ int_a^b x^2\, dx=\ frac {b^3-a^3} {3}. \] No asuma de antemano que la integral existe. La prueba de ello es parte del problema.

    2pt

    Generalizar la prueba de ejercicio~ para mostrar directamente desde Definición~ que\ [ \ int_a^b x^m\, dx=\ frac {b^ {m+1} -a^ {m+1}} {m+1} \] si\(m\) es un entero\(\ge0\).

    2pt

    Demostrar directamente desde Definition~ que\(f(x)\) es integrable on\([a,b]\) if y solo si\(f(-x)\) es integrable on\([-b,-a]\), y, en este caso, \ [\ int_a^b f (x)\, dx=\ int_ {-b} ^ {-a} f (-x)\, dx. \]

    2pt

    Dejar\(f\) estar acotado\([a,b]\) y dejar\(P\) ser una partición de\([a,b]\). Demostrar: La menor suma\(s(P)\) de\(f\) más\(P\) es el infimum del conjunto de todas las sumas de Riemann de\(f\) más\(P\).

    2pt Let\(f\) ser definido on\([a,b]\) y let\(P=\{x_0,x_1, \dots,x_n\}\) ser una partición de\([a,b]\).

    Buscar\(\underline{\int_0^1}f(x)\,dx\) y\(\overline{\int_0^1}f(x)\,dx\) si

    -.5em

    Dado que\(\int_a^be^x\,dx\) existe, evalúelo usando la fórmula\ [ 1+r+r^2+\ cdots+r^n=\ frac {1-r^ {n+1}} {1-r}\ quad (r\ ne1) \] para calcular ciertas sumas de Riemann.

    Dado que\(\int_0^b\sin x \,dx\) existe, evalúelo usando la identidad\ [ \ cos (j-1)\ theta-\ cos (j+1)\ theta=2\ sin\ theta\ sin j\ theta \] para calcular ciertas sumas de Riemann.

    Dado que\(\int_0^b\cos x\,dx\) existe, evalúelo usando la identidad\ [ \ sin (j+1)\ theta-\ sin (j-1)\ theta=2\ sin\ theta\ cos j\ theta \] para calcular ciertas sumas de Riemann.

    Mostrar que si\(g(x)=x+c\) (\(c\)=constante), entonces\(\int_a^b f(x)\,dg(x)\) existe si y solo si\(\int_a^bf(x)\,dx\) existe, en cuyo caso\ [ \ int_a^b f (x)\, dg (x) =\ int_a^bf (x)\, dx. \]

    Supongamos que\(-\infty<a<d<c<\infty\) y\ [ g (x) =\ left\ {\ casespace\ begin {array} {ll} g_1, &a<x<d,\\ g_2, &d<x<b,\ end {array}\ right. \ mbox {($g_1, g_2=$ constantes), \ quad} \] y let\(g(a)\),\(g(b)\), y\(g(d)\) ser arbitrario. Supongamos que\(f\) se define en\([a,b]\), continuo desde la derecha en\(a\) y desde la izquierda en\(b\), y continuo en\(d\). Demostrar que\(\int_a^b f(x)\,dg(x)\) existe, y encontrar su valor.

    Supongamos que\(-\infty<a=a_0<a_1<\cdots<a_p=b<\infty\), dejar\(g(x)=g_m\) (constante) encendido\((a_{m-1},a_m)\),\(1\le m\le p\), y dejar\(g(a_0)\),\(g(a_1)\),,\(g(a_p)\) ser arbitrario. Supongamos que\(f\) se define en\([a,b]\), continuo desde la derecha en\(a\) y desde la izquierda en\(b\), y continuo en\(a_1\),\(a_2\),,\(a_{p-1}\). Evaluar\(\int_a^bf(x)\,dg(x)\).

    Para el caso en el\(g\) que no sea decreciente y\(f\) esté acotado sobre\([a,b]\), defina las integrales superior e inferior de Riemann—Stieltjes de una manera análoga a Definition~.

    \ end {lista de ejercicios}

    El siguiente lema es el punto de partida para nuestro estudio de la integrabilidad de una función acotada\(f\) en un intervalo cerrado\([a,b]\).

    Te probaremos y te dejaremos el comprobante de (Ejercicio~). Primero supongamos que\(r=1\), así\(P'\) se obtiene sumando un punto\(c\) a la partición\(P=\{x_0,x_1, \dots,x_n\}\); luego\(x_{i-1}<c<x_i\) para algunos\(i\) adentro\(\{1,2, \dots,n\}\). Si\(j \ne i\), el producto\(M_j(x_j-x_{j-1})\) aparece en ambos\(S(P)\)\(S(P')\) y se cancela fuera de la diferencia\(S(P)-S(P')\). Por lo tanto, si\ [ M_ {i1} =\ sup_ {x_ {i-1}\ le x\ le c} f (x)\ mbox {\ quad y\ quad} M_ {i2} =\ sup_ {c\ le x\ le x_i} f (x), \] entonces\ [\ begin {ecuación}\ label {eq:3.2.4} \ begin {array} rcl} S (P) -S (P')\ ar=M_i (x_i-x_ {i-1}) -M_ {i1} (c-x_ {i-1}) -M_ {i2} (x_i-c) \\ [2\ jot] \ ar =( m_i-m_ {i1}) (c-x_ {i-1}) + (m_i-M_ {i2} ) (x_i-c). \ end {array} \ end {ecuación}\] Dado que implica que\ [ 0\ le m_i-M_ {ir}\ Le2m,\ quad r=1,2, \] implica que\ [ 0\ le S (P) -S (P')\ Le2m (x_i-x_ {i-1})\ Le2M\ |P\ |. \] Esto prueba para\(r=1\).

    Ahora supongamos que\(r>1\) y\(P'\) se obtiene sumando puntos\(c_1\),\(c_2\),,\(c_r\) a\(P\). Dejar\(P^{(0)}=P\) y, para\(j\ge1\), dejar\(P^{(j)}\) ser la partición de\([a,b]\) obtenida sumando\(c_j\) a\(P^{(j-1)}\). Entonces el resultado recién probado implica que\ [ 0\ le S (P^ {(j-1)}) -S (P^ {(j)})\ Le2m\ |P^ {(j-1)}\ |,\ quad 1\ le j\ le r. \]

    Añadiendo estas desigualdades y teniendo en cuenta las cancelaciones rinde\ [\ begin {ecuación}\ label {eq:3.2.5} 0\ le S (P^ {(0)}) -S (P^ {(r)})\ Le2m (\ |P^ {(0)}\ |+\ |P^ {(1)}\ |+\ cdots+\ |P^ {(r-1)} |\). \ end {ecuación}\] Dado que\(P^{(0)}=P\)\(P^{(r)}=P'\),, y\(\|P^{(k)}\|\le\|P^{(k-1)}\|\) para\(1\le k\le r-1\), implica que\ [ 0\ le S (P) -S (P')\ le 2Mr\ |P\ |, \] que es equivalente a.

    Supongamos que\(P_1\) y\(P_2\) son particiones de\([a,b]\) y\(P'\) es un refinamiento de ambos. Dejar\(P=P_1\) entrar y\(P=P_2\) entrar muestra que\ [ s (P_1)\ le s (P')\ mbox {\ quad y\ quad} S (P')\ le S (P_2). \] Ya que\(s(P')\le S(P')\), esto implica que\(s(P_1)\le S(P_2)\). Así, cada suma inferior es un límite inferior para el conjunto de todas las sumas superiores. Como\(\overline{\int_a^b}f(x)\,dx\) es el infimum de este conjunto, se deduce que\ [ s (P_1)\ le\ overline {\ int_a^b} f (x)\, dx \] para cada partición\(P_1\) de\([a,b]\). Esto significa que\(\overline{\int_a^b} f(x)\,dx\) es un límite superior para el conjunto de todas las sumas inferiores. Ya que\(\underline{\int_a^b} f(x)\,dx\) es lo supremo de este conjunto, esto implica.

    Eso lo demostramos\(\overline{\int_a^b}f(x)\,dx=\int_a^bf(x)\,dx\) y te dejamos que lo demuestres\(\underline{\int_a^b}f(x)\,dx=\int_a^bf(x)\,dx\) (Ejercicio~).

    Supongamos que\(P\) es una partición de\([a,b]\) y\(\sigma\) es una suma de Riemann de\(f\) más\(P\). Desde\ [\ begin {eqnarray*} \ overline {\ int_a^b} f (x)\, dx-\ int_a^b f (x)\, dx\ ar= \ left (\ overline {\ int_a^b} f (x)\, Dx-s (P)\ derecha) + (S (P) -\ sigma) \\ [2\ jot] &&+\ izquierda (\ sigma-\ int_a^b f (x)\ dx\ derecha), \ end {eqnarray*}\]

    la desigualdad del triángulo implica que\ [\ begin {ecuación}\ label {eq:3.2.7} \ begin {array} {rcl} \ dst {\ izquierda|\ overline {\ int_a^b} f (x)\, dx-\ int_a^b f (x)\, dx\ derecha|}\ ar\ le \ dst {\ izquierda|\ overline {\ int_a^b} f (x)\, Dx-s (P)\ derecha|+|S (P) -\ sigma|} \\\ [2\ jot] &&+\ dst {\ izquierda|\ sigma-\ int_a^b f (x)\ dx\ derecha|} . \ end {array} \ end {ecuación}\] Ahora supongamos que\(\epsilon>0\). De Definition~, hay una partición\(P_0\) de\([a,b]\) tal que\ [\ begin {ecuación}\ label {eq:3.2.8} \ overline {\ int_a^b} f (x)\, dx\ le S (P_0) < \ overline {\ int_a^b} f (x)\, dx+\ frac {\ epsilon} {3}. \ end {ecuación}\] De Definición~, hay\(\delta>0\) tal que\ [\ begin {ecuación}\ label {eq:3.2.9} \ izquierda|\ sigma-\ int_a^bf (x)\, dx\ derecha|<\ frac {\ epsilon} {3} \ end {ecuación}\] if\(\|P\|<\delta\). Ahora supongamos que\(\|P\|<\delta\) y\(P\) es un refinamiento de\(P_0\). Dado que\(S(P)\le S(P_0)\) por Lemma~, implica que\ [ \ overline {\ int_a^b} f (x)\, dx\ le S (P) < \ overline {\ int_a^b} f (x)\, dx+\ frac {\ epsilon} {3}, \] así\ [\ begin {ecuación}\ label {eq:3.2.10} \ izquierda|S (P) -\ overline {\ int_a^b} f (x)\, dx\ derecha|<\ frac {\ epsilon} {3} \ end {ecuación}\] además de. Ahora,, e implica que\ [\ begin {ecuación}\ label {eq:3.2.11} \ izquierda|\ overline {\ int_a^b} f (x)\, dx-\ int_a^b f (x)\, dx\ derecha|< \ frac {2\ epsilon} {3} +|S (P) -\ sigma| \ end {ecuación}\] por cada suma de Riemann\(\sigma\) de\(f\) más\(P\). Como\(S(P)\) es el supremo de estas sumas de Riemann (Teorem~), podemos elegir\(\sigma\) para que\ [ |S (P) -\ sigma|<\ frac {\ epsilon} {3}. \] Ahora implica que\ [ \ izquierda|\ overline {\ int_a^b} f (x)\, dx-\ int_a^b f (x)\, dx\ derecha|< \ épsilon. \] Dado que\(\epsilon\) es un número positivo arbitrario, se deduce que \ [\ overline {\ int_a^b} f (x)\, dx=\ int_a^b f (x)\, dx. \] -6.5ex6.5ex

    Te mostramos que sostiene si\(\|P\|\) es suficientemente pequeño, y te dejamos el resto de la prueba (Ejercicio~).

    La primera desigualdad en sigue inmediatamente de Definition~. Para establecer la segunda desigualdad, supongamos que\(|f(x)|\le K\) si\(a\le x\le b\). De Definition~, hay una partición\(P_0= \{x_0,x_1, \dots,x_{r+1}\}\) de\([a,b]\) tal que\ [\ begin {ecuación}\ label {eq:3.2.13} S (P_0) <\ overline {\ int_a^b} f (x)\, dx+\ frac {\ epsilon} {2}. \ end {equation}\] Si\(P\) es alguna partición de\([a,b]\),\(P'\) déjese construir a partir de los puntos de partición de\(P_0\) y\(P\). Entonces\ [\ begin {ecuación}\ label {eq:3.2.14} S (P')\ le S (P_0), \ end {ecuación}\] por Lemma~. Ya que\(P'\) se obtiene sumando como máximo\(r\) puntos a\(P\), Lemma~ implica que\ [\ begin {ecuación}\ label {eq:3.2.15} S (P')\ ge S (P) -2Kr\ |P\ |. \ end {ecuación}\] Ahora,, e implica que\ [\ begin {eqnarray*} S (P)\ ar\ le S (P') +2Kr\ |P\ |\\ \ ar\ le S (P_0) +2Kr\ |P\ |\\ &<&\ overline {\ int_a^b} f (x)\, dx+\ frac\ épsilon} {2} +2Kr\ |P\ |. \ end {eqnarray*}\] Por lo tanto, se mantiene si\ [ \ |P\ |<\ delta=\ frac {\ épsilon} {4Kr}. \] -4.5ex4.5ex

    Si\(\epsilon>0\), hay\(\delta>0\) tal que\ [\ begin {ecuación}\ label {eq:3.2.18} \ subraye {\ int_a^b} f (x)\, dx-\ épsilon<s (P)\ le S (P) < \ overline {\ int_a^b} f (x)\, dx+\ épsilon \ end {ecuación}\] if\(\|P\|<\delta\) (Lemma~). Si\(\sigma\) es una suma de Riemann de\(f\) más\(P\), entonces\ [ s (P)\ le\ sigma\ le S (P), \] así e implica que\ [ L-\ épsilon<\ sigma<l+\ épsilon \] if\(\|P\|<\delta\). Ahora Definición~ implica.

    Teoremas ~ e implican el siguiente teorema.

    El siguiente teorema traduce esto en una prueba que se puede aplicar convenientemente.

    Te dejamos a ti (Ejercicio~) demostrar que si\(\int_a^b f(x)\,dx\) existe, entonces se sostiene por\(\|P\|\) suficientemente pequeño. Esto implica que la condición declarada es necesaria para la integrabilidad. Para demostrar que es suficiente, observamos que ya que\ [ s (P)\ le\ subrayamos {\ int_a^b} f (x)\, dx\ le\ overline {\ int_a^b} f (x)\, dx\ le S (P) \] para todos\(P\), implica que\ [ 0\ le\ overline {\ int_a^b} f (x)\, dx-\ subrayado {\ int_a^b} f (x)\, dx< \ épsilon. \] Dado que\(\epsilon\) puede ser cualquier número positivo, esto implica que\ [ \ overline {\ int_a^b} f (x)\, dx=\ subrayan {\ int_a^b} f (x)\, dx. \] Por lo tanto,\(\int_a^b f(x)\,dx\) existe, por Teorem~.

    Los dos teoremas siguientes son importantes aplicaciones del Teorem~.

    Dejar\(P=\{x_0,x_1, \dots,x_n\}\) ser una partición de\([a,b]\). Ya que\(f\) es continuo on\([a,b]\), hay puntos\(c_j\) y\(c'_j\) en\([x_{j-1},x_j]\) tal que\ [f (c_j) =m_j=\ sup_ {x_ {j-1}\ le x\ le x_j} f (x) \] y\ [ f (c'_j) =m_j=\ inf_ {x_ {j-1}\ le x\ le x_j} f (x) \] (Teorem~). Por lo tanto,\ [\ begin {ecuación}\ label {eq:3.2.20} S (P) -s (P) =\ suma_ {j=1} ^n\ left [f (c_j) -f (c'_j)\ derecha] (x_j-x_ {j-1}). \ end {ecuación}\] Dado que\(f\) es uniformemente continuo en\([a,b]\) (Teorem~), hay para cada uno\(\epsilon>0\)\(\delta>0\) tal que\ [ |f (x') -f (x) |<\ frac {\ épsilon} {b-a} \] si\(x\) y\(x'\) están en\([a,b]\) y\(|x-x'|<\delta\). Si\(\|P\|<\delta\), entonces\(|c_j-c'_j|<\delta\) y, de,\ [ S (P) -s (P) <\ frac {\ epsilon} {b-a} \ sum_ {j=1} ^n (x_j-x_ {j-1}) =\ épsilon. \] Por lo tanto,\(f\) es integrable en\([a,b]\), por Teorem~.

    Dejar\(P=\{x_0,x_1, \dots,x_n\}\) ser una partición de\([a,b]\). Dado que no\(f\) es decreciente,\ [\ begin {eqnarray*} f (x_j)\ ar=m_j=\ sup_ {x_ {j-1}\ le x\ le x_j} f (x) \\\ arraytext {y}\\ f (x_ {j-1})\ ar=m_j=\ inf_ {x_ {j-1}\ le x_le j} f (x). \ end {eqnarray*}\] Por lo tanto,\ [ S (P) -s (P) =\ suma_ {j=1} ^n (f (x_j) -f (x_ {j-1})) (x_j-x_ {j-1}). \] Desde\(0<x_j-x_{j-1}\le \|P\|\) y\(f(x_j)-f(x_{j-1})\ge0\),\ [\ begin {eqnarray*} S (P) -s (P)\ ar\ le\ |P\ |\ sum_ {j=1} ^n (f (x_j) -f (x_ {j-1}))\ \ ar=\ |P\ | (f (b) -f (a)). \ end {eqnarray*}\]

    Por lo tanto,\ [ S (P) -s (P) <\ épsilon\ mbox {\ quad if\ quad} \ |P\ | (f (b) -f (a)) <\ épsilon, \] así\(f\) es integrable en\([a,b]\), por Teoremo~.

    La prueba para no aumentar\(f\) es similar.

    También utilizaremos Teorem~ en la siguiente sección para establecer propiedades de la integral. En la Sección~3.5 estudiaremos condiciones más generales para la integrabilidad.

    Ahora utilizamos los resultados de las Secciones~3.1 y 3.2 para establecer las propiedades de la integral. Probablemente estés familiarizado con la mayoría de estas propiedades, pero no con sus pruebas.

    4pt

    Cualquier suma de Riemann\(f+g\) sobre una partición\(P=\{x_0,x_1, \dots,x_n\}\) de\([a,b]\) puede escribirse como\ [ \ begin {array} {rcl} \ sigma_ {f+g}\ ar=\ dst {\ sum_ {j=1} ^n}\, [f (c_j) +g (c_j)] (x_j-x_ {j-1})\\ [2\ jot] \ ar=\ dst {\ sum_ _ {j=1} ^n}\, f (c_j) (x_j-x_ {j-1}) + \ dst {\ sum_ {j=1} ^n}\, g (c_j) (x_j-x_ {j-1})\\ [2\ jot] \ ar=\ sigma_ f+\ sigma_g, \ end {array} \] donde\(\sigma_f\) y\(\sigma_g\) son sumas de Riemann para\(f\) y\(g\). Definición~ implica que si\(\epsilon>0\) hay números positivos\(\delta_1\) y\(\delta_2\) tal que\ [\ begin {eqnarray*} \ izquierda|\ sigma_f-\ int_a^b f (x)\, dx\ derecha|\ ar<\ frac {\ epsilon} {2} \ mbox {\ quad if\ quad}\ |P\ |<\ delta_1\ \ arraytext {y}\\ \ izquierda|\ sigma_g-\ int_a^b g (x)\, dx\ derecha|\ ar<\ frac {\ epsilon} {2} \ mbox {\ quad if\ quad}\ |P\ |<\ delta_2. \ end {eqnarray*}\] Si\(\|P\|<\delta=\min(\delta_1,\delta_2)\), entonces\ [\ begin {eqnarray*} \ izquierda|\ sigma_ {f+g} -\ int_a^b f (x)\, dx-\ int_a^b g (x)\, dx\ derecha| \ ar=\ izquierda|\ izquierda (\ sigma_f-\ int_a^b (f)\, dx\ derecha) + \ izquierda (\ sigma_g-\ int_a^b g (x)\, dx\ derecha)\ derecha|\\ ar \ le\ izquierda|\ sigma_f-\ int_a^b f (x)\, dx\ derecha|+ \ izquierda|\ sigma_g-\ int _a^b g (x)\, dx\ derecha|\\ &<&\ frac {\ épsilon} {2} +\ frac {\ épsilon} {2} =\ épsilon, \ end {eqnarray*}\] así que la conclusión se desprende de Definición~.

    El siguiente teorema también se desprende de Definition~ (Ejercicio~).

    Los teoremos~ y y la inducción dan el siguiente resultado (Ejercicio~).

    Ya que\(g(x)-f(x)\ge0\), cada suma inferior de\(g-f\) sobre cualquier partición de no\([a,b]\) es negativa. Por lo tanto, \ [\ subrayan {\ int_a^b} (g (x) -f (x))\, dx\ ge0. \] Por lo tanto,\ [\ begin {ecuación}\ label {eq:3.3.2} \ begin {array} {rcl} \ dst\ int_a^b g (x)\, dx-\ int_a^b f (x)\, dx\ ar=\ dst\ int_a^b (g (x) -f (x))\, dx\ [2\ jot] \ ar=\ dst\ subrayado {\ int_a^b} (g (x) -f (x))\, dx\ ge0, \ end {array} \ end {ecuación}\] que rinde. (La primera igualdad en sigue de Teoremas~ y; la segunda, de Teorem~.)

    Dejar\(P\) ser una partición de\([a,b]\) y definir\ [\ begin {eqnarray*} m_j\ ar=\ sup\ set {f (x)} {x_ {j-1}\ le x\ le x_j},\\ m_j\ ar= \ inf\ set {f (x)} {x_ {j-1}\ le x\ le x_j},\ \ overline {M} _j\ ar=\ sup\ set {|f (x) |} {x_ {j-1}\ le x\ le x_j},\\ \ overline {m} _j\ ar=\ inf\ set {|f (x) |} {x_ {j-1}\ le x\ le x_j}. \ end {eqnarray*}\] Entonces\ [\ begin {ecuación}\ label {eq:3.3.4} \ begin {array} {rcl} \ overline {M} _j-\ overline {m} _j\ ar= \ dst\ sup\ set {|f (x) |-|f (x') |} {x_ {j-1}\ le x, x' le\ x_j}\\\ ar \ le\ dst\ sup\ set {|f (x) -f (x') |} {x_ {j-1}\ le x, x'\ le x_j}\\ \ ar=m_j-m_j. \ end {array} \ end {ecuación}\] Por lo tanto,\ [ \ overline {S} (P) -\ overline {s} (P)\ le S (P) -s (P), \] donde se asocian las sumas superior e inferior de la izquierda\(|f|\) y las de la derecha están asociadas con\(f\). Ahora supongamos eso\(\epsilon>0\). Ya que\(f\) es integrable en\([a,b]\), Teorem~ implica que hay una partición\(P\) de\([a,b]\) tal que\(S(P)-s(P)<\epsilon\). Esta desigualdad y eso implica\(\overline S(P)-\overline s(P)<\epsilon\). Por lo tanto,\(|f|\) es integrable en\([a,b]\), nuevamente por Teorem~.

    Desde\ [ f (x)\ le|f (x) |\ mbox {\ quad y\ quad} -f (x)\ le|f (x) |,\ quad a\ le x\ le b, \]

    Teoremas ~ e implican que\ [ \ int_a^b f (x)\, dx\ le\ int_a^b|f (x) |\, dx\ mbox {\ quad y} -\ int_a^b f (x)\, dx\ le\ int_a^b|f (x) |\, dx, \] lo que implica.

    Consideramos el caso donde\(f\) y no\(g\) son negativos, y te dejamos el resto de la prueba (Ejercicio~). Los subíndices\(f\),\(g\), y\(fg\) en el siguiente argumento identifican las funciones con las que están asociadas las diversas cantidades. Suponemos que\(f\) ni tampoco\(g\) es idénticamente cero encendido\([a,b]\), ya que la conclusión es obvia si uno de ellos lo es.

    Si\(P=\{x_0,x_1, \dots,x_n\}\) es una partición de\([a,b]\), entonces\ [\ begin {ecuación}\ label {eq:3.3.5} S_ {fg} (P) -s_ {fg} (p) =\ sum_ {j=1} ^n (M_ {fg, j} -m_ {fg, j}) (x_j-x_ {j-1}). \ end {ecuación}\] Dado que\(f\) y no\(g\) son negativos,\(M_{fg,j}\le M_{f,j}M_{g,j}\) y\(m_{fg,j}\ge m_{f,j}m_{g,j}\). De ahí,\ [\ begin {eqnarray*} M_ {fg, j} -m_ {fg, j}\ ar\ le M_ {f, j} M_ {g, j} -m_ {f, j} m_ {g, j}\\ [2\ jot] \ ar= (M_ {f, j} -m_ {f, j}) M_ {g, j}) M_ {g, j} m_ {f, j} (M_ {g, j} - m_ {g, j})\\ [2 \ jot]\ ar\ le m_g (M_ {f, j} -m_ {f, j}) +m_F (M_ {g, j} -m_ {g, j}), \ end {eqnarray*}\] donde\(M_f\) y\(M_g\) son límites superiores para\(f\) y\(g\) en\([a,b]\). De y la última desigualdad,\ [\ begin {ecuación}\ label {eq:3.3.6} S_ {fg} (P) -s_ {fg} (P)\ le m_g [s_f (P) -s_f (P)] +M_f [s_g (P) -s_g (P)]. \ end {ecuación}\] Ahora supongamos eso\(\epsilon>0\). Teorem~ implica que hay particiones\(P_1\) y\(P_2\) de\([a,b]\) tal manera que\ [\ begin {ecuación}\ label {eq:3.3.7} s_f (P_1) -s_f (P_1) <\ frac {\ epsilon} {2m_g}\ mbox {\ quad y\ quad} s_g (P_2) -S_g (P_2) <\ frac {\ epsilon} {2m_F}. \ end {ecuación}\] Si\(P\) es un refinamiento de ambos\(P_1\) y\(P_2\), entonces y Lemma~ implican que\ [ s_f (P) -s_f (P) <\ frac {\ épsilon} {2m_G}\ mbox {\ quad y\ quad} s_g (P) -s_g (P) <\ frac {\ épsilon} {2m_F}. \] Esto y rendimiento\ [ S_ {fg} (P) -s_ {fg} (P) <\ frac {\ epsilon} {2} +\ frac {\ épsilon} {2} =\ épsilon. \] Por lo tanto,\(fg\) es integrable en\([a,b]\), por Teorem~.

    3pt Del Teorem~,\(u\) es integrable en\([a,b]\). Por lo tanto, Teorem~ implica que la integral de la izquierda existe. Si\(m=\min\set{u(x)}{a\le x\le b}\) y\(M=\max\set{u(x)}{a\le x\le b}\) (recordar Teorem~), entonces\ [ m\ le u (x)\ le M \] y, ya que\(v(x)\ge0\),\ [ mv (x)\ le u (x) v (x)\ le Mv (x). \] Por lo tanto, Teoremas ~ e implican que 2pt\ [\ begin {ecuación}\ label {eq:3.3.9} m\ int_a^b v (x)\, dx\ le\ int_a^b u (x) v (x)\, dx\ le M\ int_a^b v (x)\, dx. \ end {ecuación}\] 2pt Esto implica que se mantiene para cualquiera\(c\) en\([a,b]\) si\(\int_a^b v(x)\,dx=0\). Si\(\int_a^b v(x)\,dx\ne0\), vamos 1pt\ [\ begin {ecuación}\ label {eq:3.3.10} \ overline {u} =\ frac {\ dst\ int_a^b u (x) v (x)\, dx} {\ dst\ int_a^bv (x)\, dx} \ end {ecuación}\] 1pt Desde\(\int_a^b v(x)\,dx>0\) en este caso (¿por qué?) , implica eso\(m\le\overline{u}\le M\), y el teorema del valor intermedio (Teorem~) implica que\(\overline{u}=u(c)\) para algunos\(c\) en\([a,b]\). Esto implica.

    1pt Si\(v(x)\equiv1\), entonces se reduce a\ [\ overline {u} =\ frac {1} { b-a}\ int_a^b u (x)\, dx,\] así\(\overline{u}\) es el promedio de\(u(x)\) más\([a,b]\). De manera más general, si\(v\) es alguna función integrable no negativa tal que\(\int_a^b v(x)\,d x\ne0\), entonces\(\overline{u}\) adentro está la {}. Teorem~ dice que una función continua asume tal promedio ponderado en algún momento de\([a,b]\).

    3pt

    Supongamos que\(\epsilon>0\). Del Teorem~, hay una partición\(P=\{x_0,x_1, \dots,x_n\}\) de\([a,b]\) tal manera que\ [\ begin {ecuación}\ label {eq:3.3.11} S (P) -s (P) =\ sum_ {j=1} ^n (m_J-m_J) (x_j-x_ {j-1}) <\ épsilon. \ end {ecuación}\] Podemos suponer que\(a_1\) y\(b_1\) son puntos de partición de\(P\), porque si no se pueden insertar para obtener un refinamiento\(P'\) tal que\(S(P')-s(P')\le S(P)-s(P)\) (Lemma~). Dejar\(a_1=x_r\) y\(b_1=x_s\). Dado que cada término en no es negativo,\ [ \ sum_ {j=r+1} ^s (m_J-m_j) (x_j-x_ {j-1}) <\ epsilon. \] Así,\(\overline{P}=\{x_r,x_{r+1}, \dots,x_s\}\) es una partición de\([a_1,b_1]\) sobre la cual las sumas superior e inferior de\(f\) satisfacer\ [ S (\ overline {P}) -s (\ overline {P}) <\ epsilon. \] Por lo tanto,\(f\) es integrable en\([a_1,b_1]\), por Teorem~.

    Te dejamos la prueba del siguiente teorema (Ejercicio~).

    Hasta el momento hemos definido\(\int_\alpha^\beta f(x)\,dx\) sólo para el caso donde\(\alpha<\beta\). Ahora definimos\ [ \ int_\ beta^\ alpha f (x)\, dx=-\ int_\ alpha^\ beta f (x)\, dx \] if\(\alpha<\beta\), y\ [ \ int_\ alpha^\ alpha f (x)\, dx=0. \] Con estas convenciones, se mantiene sin importar cuál sea el orden relativo de\(a\)\(b\), y\(c\), siempre que\(f\) sea integrable en algún intervalo cerrado que las contenga (Ejercicio~).

    Teorem~ y estas definiciones nos permiten definir una función $ F (x) =_c^x f (t), dt$, donde\(c\) es un punto arbitrario, pero fijo, en\([a,b]\).

    Si\(x\) y\(x'\) están adentro\([a,b]\), entonces\ [ F (x) -F (x') =\ int_c^x f (t)\, dt-\ int_c^ {x'} f (t)\, dt=\ int_ {x'} ^x f (t)\, dt, \] por Teorem~ y las convenciones recién adoptadas. Ya que\(|f(t)|\le K\)\((a\le t\le b)\) para alguna constante\(K\),\ [ \ izquierda|\ int_ {x'} ^x f (t)\, dt\ derecha|\ le K|x-x'|,\ quad a\ le x,\, x'\ le b \] (Teoremo~), entonces\ [ |F (x) -F (x') |\ le K|x-X'|,\ quad a\ le x,\, x'\ le b. \] -2em2em

    Consideramos el caso donde\(a<x_0<b\) y te dejamos el resto (Ejercicio~). Desde\ [ \ frac {1} {x-x_0}\ int_ {x_0} ^x f (x_0)\, dt=f (x_0), \] podemos escribir\ [ \ frac {F (x) -F (x_0)} {x-x_0} -f (x_0) =\ frac {1} {x-x_0}\ int_ {x_0} ^x [f (t) -f (x_0)]\, dt. \] De esto y Teorem~,\ [\ begin {ecuación}\ label {eq:3.3.13} \ izquierda|\ frac {F (x) -F (x_0)} {x-x_0} -f (x_0)\ derecha|\ le\ frac {1} {|x-x_0|} \ izquierda|\ int_ {x_0} ^x |f (t) -f (x_0) |\, dt\ derecha|. \ end {ecuación}\] (¿Por qué necesitamos las barras de valor absoluto fuera de la integral?) Dado que\(f\) es continuo en\(x_0\), hay para cada uno\(\epsilon>0\)\(\delta>0\) tal que\ [ |f (t) -f (x_0) |<\ épsilon\ mbox {\ quad if\ quad} |x-x_0|<\ delta \] y\(t\) está entre\(x\) y\(x_0\). Por lo tanto, de, \ [\ izquierda|\ frac {F (x) -F (x_0)} {x-x_0} -f (x_0)\ derecha|<\ épsilon \ frac {|x-x_0|} { |x-x_0|} =\ épsilon\ mbox {\ quad if\ quad} 0<|x-x_0|<\ delta. \] Por lo tanto,\(F'(x_0)=f(x_0)\).

    El siguiente teorema relaciona de otra manera la integración y diferenciación.

    Si\(P=\{x_0,x_1, \dots,x_n\}\) es una partición de\([a,b]\), entonces\ [\ begin {ecuación}\ label {eq:3.3.15} F (b) -F (a) =\ sum_ {j=1} ^n (F (x_j) -F (x_ {j-1})). \ end {ecuación}\] Del Teoremo~, hay en cada intervalo abierto\((x_{j-1},x_j)\) un punto\(c_j\) tal que\ [ F (x_j) -F (x_ {j-1}) =f (c_j) (x_j-x_ {j-1}). \]

    Por lo tanto, se puede escribir como\ [ F (b) -F (a) =\ sum_ {j=1} ^nf (c_j) (x_j-x_ {j-1}) =\ sigma, \] donde\(\sigma\) es una suma de Riemann para\(f\) más\(P\). Dado que\(f\) es integrable on\([a,b]\), hay para cada uno\(\epsilon>0\)\(\delta>0\) tal que\ [ \ izquierda|\ sigma-\ int_a^b f (x)\, dx\ derecha|<\ epsilon\ mbox {\ quad if\ quad} \ |P\ |<\ delta. \] Por lo tanto, \ [\ izquierda|F (b) -F (a) -\ int_a^b f (x)\, dx\ derecha|<\ épsilon \] para cada uno\(\epsilon>0\), lo que implica.

    Aplicar Teorem~ con\(F\) y\(f\) reemplazado por\(f\) y\(f'\), respectivamente.

    Una función\(F\) es un {} de\(f\) on\([a,b]\) si\(F\) es continuo encendido\([a,b]\) y diferenciable encendido\((a,b)\), con\ [ F' (x) =f (x),\ quad a<x<b. \] Si\(F\) es una antiderivada de\(f\) on\([a,b]\), entonces así es\(F+c\) para cualquier constante\(c\). Por el contrario, si\(F_1\) y\(F_2\) son antiderivados de\(f\) on\([a,b]\), entonces\(F_1-F_2\) es constante on\([a,b]\) (Teorema). El teorema muestra que los antiderivados pueden ser utilizados para evaluar integrales.

    La función\(F_0(x)=\int_a^x f(t)\,dt\) es continua\([a,b]\) por Teorem~, y\(F_0'(x) =f(x)\)\((a,b)\) por Teorem~. Por lo tanto,\(F_0\) es un antiderivado de\(f\) on\([a,b]\). Ahora dejemos\(F=F_0+c\) (\(c=\)constante) ser un antiderivado arbitrario de\(f\) on\([a,b]\). Entonces -2pt\ [ F (b) -F (a) =\ int_a^b f (x)\, dx+c-\ int_a^a f (x)\, dx-c=\ int_a^b f (x)\, dx. \] -2.5em2.5em

    Al aplicar este teorema, usaremos la notación familiar\ [ F (b) -F (a) =F (x)\ bigg|^b_a. \]

    Ya que\(u\) y\(v\) son continuos en\([a,b]\) (Teorem~), son integrables en\([a,b]\). Por lo tanto, Teoremas ~ e implican que la función\ [ (uv) '=u'v+uv' \] es integrable en\([a,b]\), y Teorem~ implica que \ [\ int_a^b [u (x) v' (x) +u' (x) v (x)]\, dx=u (x) v (x)\ bigg|^b_a, \] lo que implica.

    Utilizaremos Teorem~ aquí y en la siguiente sección para obtener otros resultados.

    Ya que\(f\) es diferenciable en\([a,b]\), es continuo en\([a,b]\) (Teorem~). Ya que\(g\) es continuo\([a,b]\), también lo es\(fg\) (Teorem~). Por lo tanto, Teorem~ implica que las integrales en existen. Si\ [\ begin {ecuación}\ label {eq:3.3.18} G (x) =\ int_a^x g (t)\, dt, \ end {ecuación}\] entonces\(G'(x)=g(x),\ a<x<b\) (Teoremo~). Por lo tanto, Teorem~ con\(u=f\) y\(v=G\) rinde\ [\ begin {ecuación}\ label {eq:3.3.19} \ int_a^b f (x) g (x)\, dx=f (x) G (x)\ bigg|^b_a-\ int_a^b f' (x) G (x)\, dx. \ end {ecuación}\] Como no\(f'\) es negativo y\(G\) es continuo, el Teoremo~ implica que\ [\ begin {ecuación}\ label {eq:3.3.20} \ int_a^b f' (x) G (x)\, dx=G (c)\ int_a^b f' (x)\, dx \ end {ecuación}\]

    para algunos\(c\) en\([a,b]\). De Corolario ~,\ [ \ int_a^b f' (x)\, dx=f (b) -f (a). \] A partir de esto y, se puede reescribir como \ [\ int_a^b f' (x) G (x)\, dx =( f (b) -f (a))\ int_a^c g (x)\, dx. \] Sustituyendo esto en y señalando que\(G(a)=0\) produce\ [\ begin {eqnarray*} \ int_a^b f (x) g (x)\, dx\ ar=f (b)\ int_a^b g (x)\, dx- (f (b) -f (a)) \ int_a^c g (x)\, dx,\ \ ar=f (a)\ int_a^c g (x)\, dx+f (b)\ izquierda (\ int_a^b g (x)\, dx-\ int_c^a g (x)\, dx\ derecha)\ \ ar=f (a)\ int_a^c g (x)\, dx+f (b)\ int_c^b g (x)\, dx. \ end {eqnarray*}\] -2.5em2.5em

    -.4em El siguiente teorema sobre el cambio de variable es útil para evaluar integrales.

    Ambas integrales en existen: la de la izquierda por Teorem~, la de la derecha por Teoremas~ y y la continuidad de\(f(\phi(t))\). Por Teorem~, la función\ [ F (x) =\ int_a^x f (y)\, dy \] es una antiderivada de\(f\) on\([a,b]\) y, por lo tanto, también en el intervalo cerrado con puntos finales\(\alpha\) y\(\beta\). De ahí, por Teoremo~,\ [\ begin {ecuación}\ label {eq:3.3.22} \ int_\ alpha^\ beta f (x)\, dx=F (\ beta) -F (\ alpha). \ end {ecuación}\] Por la regla de la cadena, la función\ [ G (t) =F (\ phi (t)) \]

    es una antiderivada de\(f(\phi(t))\phi'(t)\) on\([c,d]\), y Teorem~ implica que\ [\ begin {eqnarray*} \ int_c^d f (\ phi (t))\ phi' (t)\, dt\ ar= G (d) -G (c) =F (\ phi (d)) -F (\ phi (c))\ \ ar=F (\ beta) -F (\ alpha). \ end {eqnarray*}\] Comparando esto con rendimientos.

    Estos ejemplos ilustran dos formas de usar Teorem~. En Ejemplo~ evaluamos el lado izquierdo de transformándolo al lado derecho con una sustitución adecuada\(x=\phi(t)\), mientras que en Ejemplo~ evaluamos el lado derecho de reconociendo que podría obtenerse del lado izquierdo por una sustitución adecuada.

    El siguiente teorema muestra que la regla para el cambio de variable sigue siendo válida bajo supuestos más débiles sobre\(f\) si\(\phi\) es monótona.

    Consideramos el caso donde no\(f\) es negativo y\(\phi\) no disminuye, y te dejamos el resto de la prueba (Ejercicios~ y).

    Primero supongamos que\(\phi\) va en aumento. Mostramos primero que\ [\ begin {ecuación}\ label {eq:3.3.24} \ overline {\ int^b_a} f (x)\, dx=\ overline {\ int^d_c} f (\ phi (t))\ phi' (t)\, dt. \ end {ecuación}\] Dejar\(\overline{P}=\{t_0,t_1, \dots,t_n\}\) ser una partición de\([c,d]\) y\(P=\{x_0,x_1, \dots,x_n\}\) con\(x_j=\phi(t_j)\) ser la partición correspondiente de\([a,b]\). Definir\ [\ begin {eqnarray*} u_j\ ar=\ sup\ set {\ phi' (t)} {t_ {j-1}\ le t\ le t_j},\\ u_j\ ar=\ inf\ set {\ phi' (t)} {t_ {j-1}\ le t\ le t_j},\\ m_j\ ar=\ sup set\ {f (x)} {x_ {j-1}\ le x\ le x_j},\\ \ arraytext {y}\\ \ overline {M} _j\ ar=\ sup\ set {f (\ phi (t))\ phi' (t)} {t_ {j-1}\ le t\ le t_j}. \ end {eqnarray*}\] Ya que\(\phi\) va en aumento,\(u_j\ge0\). Por lo tanto,\ [ 0\ le u_j\ le\ phi' (t)\ le u_j,\ quad t_ {j-1}\ le t\ le t_j. \] Dado que no\(f\) es negativo, esto implica que\ [ 0\ le f (\ phi (t)) u_j\ le f (\ phi (t))\ phi' (t)\ le f (\ phi (t)) u_j,\ quad t_ {j-1}\ le t\ le t_j. \] Por lo tanto,\ [ m_ju_j\ le \ overline {M} _j\ le m_ju_j, \]

    lo que implica que\ [\ begin {ecuación}\ label {eq:3.3.25} \ overline {M} _j=M_j\ rho_j, \ end {ecuación}\] donde\ [\ begin {ecuación}\ label {eq:3.3.26} u_j\ le\ rho_j\ le u_j. \ end {ecuación}\]

    Ahora considere las sumas superiores\ [\ begin {ecuación}\ label {eq:3.3.27} \ overline {S} (\ overline {P}) =\ suma_ {j=1} ^n\ overline {M} _j (t_j-t_ {j-1}) \ mbox {\ quad y\ quad} S (P) =\ suma_ {j=1} ^n m_j (x_j x_ {j-1}). \ end {ecuación}\] Del teorema del valor medio,\ [\ begin {ecuación}\ label {eq:3.3.28} x_j-x_ {j-1} =\ phi (t_j) -\ phi (t_ {j-1}) =\ phi' (\ tau_j) (t_j-t_ {j-1}), \ end {ecuación}\] donde\(t_{j-1}<\tau_j<t_j\), así\ [\ begin ecuación}\ label {eq:3.3.29} u_j\ le\ phi' (\ tau_j)\ le u_j. \ end {ecuación}\] De,, y,\ [\ begin { ecuación}\ label {eq:3.3.30} \ overline {S} (\ overline {P}) -S (P) =\ suma_ {j=1} ^n m_J (\ rho_j-\ phi' (\ tau_j)) (t_j-t_ {j-1}). \ end {ecuación}\]

    Ahora supongamos que\(|f(x)|\le M\),\(a\le x\le b\). Entonces,, e implica que\ [ \ izquierda|\ overline {S} (\ overline {P}) -S (P)\ derecha|\ le M\ suma_ {j=1} ^n (u_j-u_j) (t_j-t_ {j-1}). \] La suma de la derecha es la diferencia entre las sumas superior e inferior de\(\phi'\) más\(\overline{P}\). Dado que\(\phi'\) es integrable en\([c,d]\), esto se puede hacer tan pequeño como nos plazca eligiendo\(\|\overline{P}\|\) suficientemente pequeño (Ejercicio~).

    De,\(\|P\|\le K\|\overline{P}\|\) si\(|\phi'(t)|\le K\),\(c\le t\le d\). De ahí que Lemma~ implica que\ [\ begin {ecuación}\ label {eq:3.3.31} \ izquierda| S (P) -\ overline {\ int_a^b} f (x)\, dx\ derecha|<\ frac {\ epsilon} {3} \ mbox {\ quad y\ quad}\ izquierda|\ overline {S} (\ overline {P}) -\ overline {\ int_c^d} f (\ phi (t))\ phi' (t)\, dt\ derecha|<\ frac {\ epsilon} {3} \ end {ecuación}\] si\(\|\overline{P}\|\) es suficientemente pequeña. Ahora\ [\ begin {eqnarray*} \ izquierda|\ overline {\ int_a^b} f (x)\, dx-\ overline {\ int_c^d} f\ izquierda (\ phi (t)\ derecha)\ phi' (t)\, dt\ derecha|\ ar\ le\ izquierda|\ overline {\ int_a^b} f (x)\, Dx-s (P)\ derecha| +|S (P) -\ overline {S} (\ overline {P}) |\\ &&+\ izquierda| \ overline {S} (\ overline {P}) -\ overline {\ int_c^d} f (\ phi (t)) \ phi' (t)\, dt\ derecha|. \ end {eqnarray*}\] Escogiendo\(\overline{P}\) así que\(|S(P)-\overline{S}(\overline{P}|< \epsilon/3\) además de rendimientos\ [ \ izquierda|\ int_a^b f (x)\, dx-\ int_c^d f (\ phi (t))\ phi' (t)\, dt \ right|<\ épsilon. \] Dado que\(\epsilon\) es un número positivo arbitrario, esto implica.

    Si\(\phi\) es no decreciente (en lugar de aumentar), puede suceder que\(x_{j-1}=x_j\) para algunos valores de\(j\); sin embargo, esto no es una complicación real, ya que simplemente significa que algunos términos en\(S(P)\) desaparecen.

    Al aplicar a\(-f\), inferimos que\ [\ begin {eqnarray} \ subrayamos {\ int_a^b} f (x)\, dx\ ar=\ subrayado {\ int_c^d} f (\ phi (t))\ phi' (t)\, dt,\ label {eq:3.3.32}\\ [2\ jot] \ arraytext {since}\ nonumber\\ \ overline {\ int_a^b} (-f) (x)\, dx\ ar=-\ subrayado {\ int_a^b} f (x)\, dx\ nonumber\\ \ arraytext {y}\ nonumber\\ \ overline {\ int_c^d} (-f (\ phi (t)\ phi' (t))\, dt\ ar=-\ subrayado {\ int_c^d} f (\ phi (t))\ phi' (t)\, dt. \ nonumber \ end {eqnarray}\]

    Ahora supongamos que\(f\) es integrable en\([a,b]\). Entonces\ [ \ subrayar {\ int_a^b} f (x)\, dx=\ overline {\ int_a^b} f (x)\, dx= \ int_a^bf (x)\, dx, \] por Teoremo~. A partir de esto,, y, \ [\ subrayan {\ int^d_c} f (\ phi (t))\ phi' (t)\, dt= \ overline {\ int^d_c} f (\ phi (t))\ phi' (t)\, dt= \ int^b_a f (x)\, dx. \] Esto y Teorem~ (aplicado a\(f(\phi(t))\phi'(t)\)) implican que\(f(\phi(t))\phi'(t)\) es integrable on\([c,d]\) y\ [\ begin {ecuación}\ label {eq:3.3.33} \ int_a^b f (x)\, dx=\ int_c^d f (\ phi (t))\ phi' (t)\, dt. \ end {equation}\] Un argumento similar muestra que si\(f(\phi(t))\phi'(t)\) es integrable on\([c,d]\), entonces\(f\) es integrable on\([a,b]\), y se mantiene.

    \ begin {exerciselist}

    Demostrar teoremo~. 4pt

    Demostrar teoremo~.

    4pt

    ¿Puede\(|f|\) ser integrable en\([a,b]\) si no lo\(f\) es? 4pt

    Completar la prueba de Teoremo~.

    4pt

    Demostrar: Si\(f\) es integrable en\([a,b]\) y\(|f(x)|\ge\rho>0\) para\(a\le x\le b\), entonces\(1/f\) es integrable en\([a,b]\)

    Supongamos que\(f\) es integrable on\([a,b]\) y define\ [ f^+ (x) =\ left\ {\ casespace\ begin {array} {ll} f (x) &\ mbox {\ quad if $f (x)\ ge0, $}\\ [2\ jot] 0&\ mbox {\ quad if $f (x) <0$,}\ end {array}\ right. \ mbox {y\ quad} f^- (x) =\ left\ {\ casespace\ begin {array} {ll} 0& \ mbox {\ quad si $f (x)\ ge0$},\\ [2\ jot] f (x) &\ mbox {\ quad si $f (x) <0$. \ quad} \ end {array}\ derecho. \] Mostrar eso\(f^+\) y\(f^-\) son integrables on\([a,b]\), y\ [ \ int_a^b f (x)\, dx=\ int_a^b f^+ (x)\, dx+\ int_a^b f^- (x)\, dx. \]

    Encontrar el promedio ponderado\(\overline{u}\) de\(u(x)\) más\([a,b]\) con respecto a\(v\), y encontrar un punto\(c\) en\([a,b]\) tal que\(u(c)=\overline{u}\).

    Demostrar teoremo~.

    Mostrar que\ [ \ int_a^c f (x)\, dx=\ int_a^b f (x)\, dx+\ int_b^c f (x)\, dx \] para todos los ordenamientos relativos posibles de\(a\),\(b\), y\(c\), siempre que\(f\) sea integrable en un intervalo cerrado que los contenga.

    Demostrar: Si\(f\) es integrable on\([a,b]\) y\(a=a_0<a_1<\cdots<a_n=b\), entonces \ [\ int_a^bf (x)\, dx=\ int_ {a_0} ^ {a_1} f (x)\, dx+\ int_ {a_1} ^ {a_2} f (x)\, dx +\ cdots+\ int_ {a_ {n-1}} ^ {a_n} f (x)\, dx. \]

    Supongamos que\(f\) es continuo\([a,b]\) y\(P=\{x_0,x_1, \dots,x_n\}\) es una partición de\([a,b]\). Demuestre que hay una suma Riemann de\(f\) más de\(P\) eso {}\(\int_a^b f(x)\,dx\).

    Supongamos que\(f'\) existe y\(|f'(x)|\le M\) sigue\([a,b]\). Mostrar que cualquier suma\(\sigma\) de Riemann\(f\) sobre cualquier partición\(P\) de\([a,b]\) satisface\ [ \ izquierda|\ sigma-\ int_a^b f (x)\, dx\ derecha|\ le M (b-a)\ |P\ |. \]

    Demostrar: Si\(f\) es integrable y\(f(x)\ge0\)\([a,b]\) encendido\(\int_a^b f(x)\,dx\ge0\), entonces, con estricta desigualdad si\(f\) es continuo y positivo en algún momento en\([a,b]\).

    Completar la prueba de Teoremo~.

    Teoremas de Estado análogos a Teoremas~ y para la función\ [ G (x) =\ int_x^c f (t)\, dt, \] y mostrar cómo se pueden obtener tus teoremas de ellos.

    El símbolo\(\int f(x)\,dx\) denota una antiderivada de\(f\). Un análogo plausible del Teoremo~ declararía que si\(f\) y\(g\) tienen antiderivadas activadas\([a,b]\), entonces también lo hace\(f+g\), lo cual es cierto, y \ [\ int (f+g) (x)\, dx=\ int f (x)\, dx+\ int g (x)\, dx. \ eqno {\ rm (A)} \] Sin embargo, esto no es cierto en el sentido habitual.

    (Ver Ejercicio~.) Formular una interpretación válida de la relación\ [ \ int (cf) (x)\, dx=c\ int f (x)\, dx\ quad (c\ ne0). \] ¿Su interpretación es válida si\(c=0\)?

    Además de los supuestos del Teorem~, supongamos que\(f(a)=0\),\(f\not\equiv0\), y\(g(x)>0\)\((a<x<b)\). Demostrar que solo hay un punto\(c\) en\([a,b]\) con la propiedad señalada en Teorem~.

    Suponiendo que Teorem~ es cierto bajo la suposición adicional que no\(f\) es negativa en\([a,b]\), mostrar que es cierto sin esta suposición.

    Suponiendo que la conclusión del Teorem~ es verdadera si no\(\phi\) es decreciente, mostrar que es verdad si no\(\phi\) es creciente.

    Supongamos que\(g'\) es integrable y\(f\) es continuo en\([a,b]\). Demostrar que\(\int_a^b f(x)\,dg(x)\) existe y es igual\(\int_a^b f(x)g'(x)\,dx\).

    Supongamos\(f\) y\(g''\) están acotados y\(fg'\) es integrable en\([a,b]\). Demostrar que\(\int_a^b f(x)\,dg(x)\) existe y es igual\(\int_a^b f(x)g'(x)\,dx\).

    \ end {lista de ejercicios}

    Hasta ahora hemos confinado nuestro estudio de las funciones integrales a delimitadas en intervalos cerrados finitos. Esto fue por buenas razones:

    En esta sección ampliamos la definición de integral para incluir casos donde\(f\) está sin límites o el intervalo no está acotado, o ambos.

    Decimos\(f\) es {} en un intervalo\(I\) si\(f\) es integrable en cada subintervalo cerrado finito de\(I\). Por ejemplo,\ [ f (x) =\ sin x \] es integrable localmente en\((-\infty,\infty)\);\ [ g (x) =\ frac {1} {x (x-1)} \] es integrable localmente en\((-\infty,0)\),\((0,1)\), y\((1,\infty)\); y\ [ h (x) =\ sqrt {x} \] es integrable localmente en\([0,\infty)\).

    El límite en siempre existe si\([a,b)\) es finito y\(f\) es localmente integrable y limitado en\([a,b)\). En este caso, Definiciones~ y asignar el mismo valor a\(\int_a^b f(x)\,dx\) no importa cómo\(f(b)\) se defina (Ejercicio~). Sin embargo, el límite también puede existir en los casos en que\(b=\infty\) o\(b<\infty\) y\(f\) es ilimitado como\(x\) aproximaciones\(b\) desde la izquierda. En estos casos, Definition~ asigna un valor a una integral que no existe en el sentido de Definición~, y\(\int_a^b f(x)\, dx\) se dice que es un {} que {} al límite en. También decimos en este caso que\(f\) es {}\([a,b)\) y que\(\int_a^b f(x)\,dx\) {}. Si el límite en no existe (finito), decimos que la integral impropia\(\int_a^b f(x)\,dx\) {}, y\(f\) es {}\([a,b)\). En particular, si\(\lim_{c\to b-}\int_a^c f(x)\,dx=\pm\infty\), decimos eso\(\int_a^b f(x)\,dx\) {}\(\pm\infty\), y escribimos\ [ \ int_a^b f (x)\, dx=\ infty\ mbox {\ quad o\ quad}\ int_a^b f (x)\, dx=- \ infty, \] cualquiera que sea el caso.

    Comentarios similares se aplican a las dos definiciones siguientes.

    La existencia y el valor de\(\int_a^b f(x)\,dx\) según Definición~ no dependen de la elección particular de\(\alpha\) in\((a,b)\) (Ejercicio~).

    Cuando deseamos distinguir entre integrales impropias e integrales en el sentido de Definición~, llamaremos a estas últimas {}.

    Al exponer y probar teoremas sobre integrales impropias, consideraremos integrales del tipo introducido en Definition~. Resultados similares se aplican a las integrales de Definiciones~ y. Te dejamos a ti formularlos y utilizarlos en los ejemplos y ejercicios según surja la necesidad.

    Si\(a<c<b\), entonces\ [\ begin {eqnarray*} \ int_a^c (c_1f_1+c_2f_2+\ cdots+c_nf_n) (x)\, dx\ ar=c_1\ int_a^c f_1 (x)\, dx +c_2\ int_a^c f_2 (x)\, dx\ \ ar {+\ cdots+c_n\ int_a^c f_n (x)\, dx, \ end {eqnarray*}\] por Teoremo~. Dejar\(c\to b-\) arroja el resultado declarado.

    La teoría de integrales impropias de funciones no negativas es particularmente simple.

    2pt Dado que no\(F\) es decreciente en\([a,b)\), Teorem~

    implica la conclusión.

    A menudo escribimos 1pt\ [ \ int_a^b f (x)\, dx<\ infty \] 2pt

    para indicar que converge una integral inadecuada de una función no negativa. Teorem~ justifica esta convención, ya que afirma que una integral divergente de este tipo sólo puede divergir a\(\infty\). Del mismo modo, si\(f\) es no positivo y\(\int_a^b f(x)\,dx\) converge, escribimos 1pt \ [\ int_a^b f (x)\, dx>-\ infty \] 2pt

    porque una integral divergente de este tipo sólo puede divergir a\(-\infty\). (Para ver esto, aplicar Teorem~ a\(-f\).) Estas convenciones no se aplican a integrales inadecuadas de funciones que asumen valores tanto positivos como negativos en\((a,b)\), ya que pueden divergir sin divergir a\(\pm\infty\).

    4pt

    La suposición implica que \ [\ int_a^x f (t)\, dt\ le\ int_a^x g (t)\, dt,\ quad a\ le x<b \] (Teoremo~), así que \ [\ sup_ {a\ le x<b}\,\ int_a^x f (t)\, dt\ le sup_ {a\ le x\ le b}\,\ int_a^x g (t)\, dt. \] Si\(\int_a^b g(x)\,dx<\infty\), el lado derecho de esta desigualdad es finito por Teoremo~, entonces el lado izquierdo es también. Esto implica que\(\int_a^b f(x)\,dx<\infty\), nuevamente por Teorem~.

    La prueba es por contradicción. Si\(\int_a^bg(x)\,dx<\infty\), entonces

    implica eso\(\int_a^bf(x)\,dx<\infty\), contradiciendo la suposición de que\(\int_a^bf(x)\,dx=\infty\).

    La prueba de comparación es particularmente útil si el integrando de la integral inadecuada es complicado pero se puede comparar con una función que es fácil de integrar.

    Si alguna función (no necesariamente no negativa)\(f\) se puede integrar localmente en\([a,b)\), entonces\ [ \ int_a^c f (x)\, dx=\ int_a^ {a_1} f (x)\, dx+\ int_ {a_1} ^c f (x)\, dx \] si\(a_1\) y\(c\) están en\([a,b)\). Ya que\(\int_a^{a_1}f(x)\,dx\) es una integral propiamente dicha, al dejar\(c\to b-\) concluir que si alguna de las integrales impropias\(\int_a^b f(x)\,dx\) y\(\int_{a_1}^b f(x)\,dx\) converge entonces también lo hace la otra, y en este caso \ [\ int_a^b f (x)\, dx=\ int_a^ {a_1} f (x)\, dx+\ int_ {a_1} ^b f (x)\, dx. \]

    Esto significa que cualquier teorema que implique convergencia o divergencia de una integral impropia\(\int_a^b f(x)\,dx\) en el sentido de Definición~ sigue siendo válido si sus hipótesis se satisfacen en un subintervalo\([a_1,b)\) de\([a,b)\) más que en todos\([a,b)\). Por ejemplo, Teorem~ sigue siendo válido si se sustituye por\ [ 0\ le f (x)\ le g (x),\ quad a_1\ le x<b, \] donde\(a_1\) hay algún punto en\([a,b)\).

    A partir de esto, se puede ver que si\(f(x)\ge0\) en algún subintervalo\([a_1,b)\) de\([a,b)\), pero no necesariamente para todos\(x\) en\([a,b)\), todavía podemos usar la convención introducida anteriormente para funciones positivas; es decir, podemos escribir\(\int_a^bf(x)\,dx<\infty\) si la integral inadecuada converge o\(\int_a^bf(x)\,dx=\infty\) si diverge.

    De, hay un punto\(a_2\) en\([a_1,b)\) tal que\ [ 0<\ frac {M} {2} <\ frac {f (x)} {g (x)} <\ frac {3M} {2},\ quad a_2\ le x<b, \] y por lo tanto\ [\ begin {ecuación}\ label {eq:3.4.4} \ frac {M} {2} g (x) <f (x) <\ frac {3M} {2} g (x),\ quad a_2\ le x<b. \ end {ecuación}\] Teorem~ y el primera desigualdad en implica que\ [ \ int_ {a_2} ^b g (x)\, dx<\ infty\ mbox {\ quad if\ quad}\ int_ {a_2} ^b f (x)\, dx<\ infty. \]

    Teorem~ y la segunda desigualdad en implican que\ [ \ int_ {a_2} ^b f (x)\, dx<\ infty\ mbox {\ quad if\ quad}\ int_ {a_2} ^b g (x)\, dx<\ infty. \] Por lo tanto,\(\int_{a_2}^b f(x)\,dx\) y\(\int_{a_2}^b g(x)\,dx\) convergen o divergen juntos, y en este último caso deben divergir a\(\infty\), ya que sus integrands son no negativos (Teorem~).

    Si\(M=\infty\), hay un punto\(a_2\) en\([a_1,b)\) tal que\ [ f (x)\ ge g (x),\ quad a_2\ le x\ le b, \] así Teoremo~

    implica eso\(\int_a^bf(x)\,dx=\infty\).

    Si\(M=0\), hay un punto\(a_2\) en\([a_1,b)\) tal que\ [ f (x)\ le g (x),\ quad a_2\ le x\ le b, \] así Teoremo~

    implica eso\(\int_a^bf(x)\,dx<\infty\).

    Las hipótesis del Teorem~ y no implican eso\(\int_a^bf(x)\,dx\) y\(\int_a^bg(x)\,dx\) necesariamente convergen o divergen juntas. Por ejemplo, si\(b=\infty\), entonces\(f(x)=1/x\) y\(g(x)=1/x^2\) satisfacer las hipótesis del Teoremo~ , mientras\(f(x)=1/x^2\) y\(g(x)=1/x\) satisfacer las hipótesis del Teorem~

    . Sin embargo,\(\int_1^\infty 1/x\,dx=\infty\), mientras\(\int_1^\infty 1/x^2\,dx<\infty\).

    1em

    {}

    1em

    1em

    Si\ [ g (x) =|f (x) |-f (x), \]

    entonces\ [ 0\ le g (x)\ le2|f (x) | \] y\(\int_a^b g(x)\,dx<\infty\), debido al Teoremo~ y la integrabilidad absoluta de\(f\). Ya que\ [ f=|f|-g, \] Teorem~ implica que\(\int_a^b f(x)\,dx\) converge.

    -.15em Decimos que\(f\) es {} a\(b\negthickspace-(=\infty\mbox{\quad if\quad} b=\infty)\) si\(f\) se define encendido\([a,b)\) y no cambia de signo en algún subintervalo\([a_1,b)\) de\([a,b)\). Si\(f\) cambia el signo en cada subintervalo,\(f\) es {} en\(b-\). Para una función que es localmente integrable on\([a,b)\) y no oscilatoria at\(b-\), convergencia y convergencia absoluta de\(\int_a^b f(x)\,dx\) cantidad a lo mismo (Ejercicio~), entonces la convergencia absoluta no es un concepto interesante en relación con tales funciones. Sin embargo, una función oscilatoria puede ser integrable, pero no absolutamente integrable, on\([a,b)\), como muestra el siguiente ejemplo. Entonces decimos que\(f\) es {} integrable encendido\([a,b)\), y eso\(\int_a^b f(x)\,dx\) converge {}.

    El método utilizado en Ejemplo~ es un caso especial de la siguiente prueba para la convergencia de integrales inadecuadas.

    La función continua\(fg\) es integrable localmente en\([a,b)\). Integración por partes rinde\ [\ begin {ecuación}\ label {eq:3.4.10} \ int_a^c f (x) g (x)\, dx=F (c) g (c) -\ int_a^c F (x) g' (x)\, dx,\ quad a\ le c<b. \ end {ecuación}\] Teorem~ implica que la integral de la derecha es absolutamente como\(c\to b-\), ya que\(\int_a^b |g'(x)|\,dx<\infty\) por suposición, y\ [ |F (x) g' (x) |\ le m|g' (x) | , \] donde\(M\) es un límite superior para\(|F|\) on\([a,b)\). Por otra parte, y la amplitud de\(F\) implica eso\(\lim_{c\to b-}F(c)g(c)=0\). Dejar\(c\to b-\) entrar rendimientos\ [ \ int_a^b f (x) g (x)\, dx=-\ int_a^b F (x) g' (x)\, dx, \] donde la integral de la derecha converge absolutamente.

    La prueba de Dirichlet es útil solo si\(f\) es oscilatoria at\(b-\), ya que se puede demostrar que si\(f\) es no oscilatoria en\(b-\) y\(F\) está delimitada en\([a,b)\), entonces\(\int_a^b |f(x)g(x)|\,dx<\infty\) si solo\(g\) es integrable localmente y acotada en\([a,b)\) (Ejercicio~).

    El método utilizado en Ejemplo~ es un caso especial de la siguiente prueba para la divergencia de integrales inadecuadas.

    La prueba es por contradicción. Dejemos\(f=uv\) y\(g=1/v\), y supongamos que eso\(\int_a^bu(x)v(x)\,dx\) converge. Entonces\(f\) tiene el antiderivado acotado\(F(x)=\int_a^xu(t)v(t)\,dt\) encendido\([a,b)\),\(\lim_{x\to\infty}g(x)=0\) y\(g'=-v'/v^2\) es absolutamente integrable en\([a,b)\). Por lo tanto, Teorem~ implica que\(\int_a^b u(x)\,dx\) converge, una contradicción.

    Si la prueba de Dirichlet demuestra que eso\(\int_a^b f(x)g(x)\,dx\) converge, queda la cuestión de si converge absoluta o condicionalmente. El siguiente teorema a veces responde a esta pregunta. Su prueba se puede modelar según el método de Ejemplo~ (Ejercicio~). La idea de una secuencia infinita, que discutiremos en la Sección~4.1, entra en el enunciado de este teorema. Suponemos que recuerda suficientemente bien el concepto del cálculo para entender el significado del teorema.

    -.4em El siguiente teorema nos permite investigar una integral impropia transformándola en otra cuya convergencia o divergencia se conoce. Se desprende del Teorem~ y Definiciones~,, y. Omitimos la prueba.

    En la Sección~3.2 encontramos condiciones necesarias y suficientes para la existencia de la integral propia de Riemann, y en la Sección~3.3 las utilizamos para estudiar las propiedades de la integral. Sin embargo, resulta incómodo aplicar estas condiciones a una función específica y determinar si es integrable, ya que requieren cálculos de sumas superiores e inferiores e integrales superiores e inferiores, lo que puede resultar difícil. El principal resultado de esta sección es un criterio de integrabilidad debido a que no requiere cómputos, sino que tiene que ver con lo mal que puede ser una función discontinua y aún así ser integrable.

    Destacamos que nuevamente estamos considerando integrales adecuadas de funciones delimitadas en intervalos finitos.

    Para un fijo\(x\) en\((a,b)\),\(W_f(x-h,x+h)\) es una función no negativa y no decreciente de\(h\) for\(0<h<\min(x-a,b-x)\); por lo tanto,\(w_f(x)\) existe y no es negativo, por Teorem~. Argumentos similares se aplican a\(w_f(a)\) y\(w_f(b)\).

    Supongamos que\(a<x_0<b\). Primero, supongamos que\(w_f(x_0)=0\) y\(\epsilon>0\). Entonces\ [ w_f [x_0-h, x_0+h] <\ épsilon \] para algunos\(h>0\), así que\ [ |f (x) -f (x') |<\ épsilon\ mbox {\ quad if\ quad} x_0-h\ le x, x'\ le x_0+h. \] Dejando\(x'=x_0\), concluimos que\ [ |f (x) -f (x_0 _0) |<\ épsilon\ mbox {\ quad if\ quad} |x-x_0|<h. \] Por lo tanto,\(f\) es continuo en \(x_0\).

    Por el contrario, si\(f\) es continuo en\(x_0\) y\(\epsilon>0\), hay\(\delta>0\) tal que\ [ |f (x) -f (x_0) |<\ frac {\ epsilon} {2}\ mbox {\ quad y\ quad} |f (x') -f (x_0) |< \ frac {\ epsilon} {2} \] if\(x_0-\delta\le x\),\(x'\le x_0+\delta\). Del triángulo desigualdad,\ [ |f (x) -f (x') |\ le|f (x) -f (x_0) |+|f (x') -f (x_0) |<\ épsilon, \] así que\ [ w_f [x_0-h, x_0+h]\ le\ épsilon\ mbox {\ quad if\ quad} h<\ delta; \] por lo tanto,\(w_f(x_0)=0\). Argumentos similares se aplican si\(x_0=a\) o\(x_0=b\).

    Utilizamos el teorema de Heine—Borel (Teorem~). Si\(w_f(x)<\epsilon\), hay\(h_x>0\) tal que\ [\ begin {ecuación}\ label {eq:3.5.1} |f (x') -f (x”) |<\ épsilon \ end {ecuación}\]

    if\ [\ begin {ecuación}\ label {eq:3.5.2} x-2h_x<x', x"<x+2h_x\ mbox {\ quad y\ quad} x', x” \ in [a, b]. \ end {equation}\] Si\(I_x=(x-h_x,x+h_x)\), entonces la colección\ [ {\ mathcal H} =\ set {i_x} {a\ le x\ le b} \] es una cubierta abierta de\([a,b]\), entonces el teorema de Heine—Borel implica que hay finitamente muchos puntos\(x_1\),\(x_2\),,\(x_n\) en\([a,b]\) tal que\(I_{x_1}\),\(I_{x_2}\),, \(I_{x_n}\)cubierta\([a,b]\). Vamos\ [ h=\ min_ {1\ le i\ le n} h_ {x_i} \] y supongamos que\([a_1,b_1]\subset [a,b]\) y\(b_1-a_1<h\). Si\(x'\) y\(x''\) están adentro\([a_1,b_1]\), entonces\(x'\in I_{x_r}\) para algunos\(r\ (1\le r \le n)\), así que\ [ |x'-x_r|<h_ {x_r}. \] Por lo tanto,\ [\ begin {eqnarray*} |x” -x_r|\ ar\ le |x” -x'|+|x'-x_r| <b_1-a_1+h_ {x_r}\\ &<&h+h_ {x_r} \ le2h_ {x_r}. \ end {eqnarray*}\] Así, cualesquiera dos puntos\(x'\) y\(x''\) en\([a_1,b_1]\) satisfacer con\(x=x_r\), por lo que también satisfacen. Por lo tanto,\(\epsilon\) es un límite superior para el conjunto \ [\ set {|f (x') -f (x”) |} {x', x "\ in [a_1, b_1]}, \] que tiene el supremo\(W_f[a_1,b_1]\). De ahí,\(W_f[a_1,b_1]\le\epsilon\).

    En lo siguiente,\(L(I)\) se encuentra la longitud del intervalo\(I\).

    Primero mostramos que\(E_\rho\) está cerrado. Supongamos que\(x_0\) es un punto límite de\(E_\rho\). Si\(h>0\), hay un\(\overline{x}\) de\(E_\rho\) adentro\((x_0-h,x_0+h)\). Ya que\([\overline{x}-h_1,\overline{x}+h_1] \subset [x_0-h,x_0+h]\) para suficientemente pequeños\(h_1\) y\(W_f[\overline{x}-h_1,\overline{x}+h_1]\ge\rho\), se deduce que\(W_f[x_0-h,x_0+h]\ge\rho\) para todos\(h>0\). Esto implica que\(x_0\in E_\rho\), así\(E_\rho\) está cerrado (Corolary~).

    Ahora vamos a demostrar que la condición declarada es necesaria para la integrabilidad. Supongamos que no se cumple la condición; es decir, hay a\(\rho>0\) y\(\delta>0\) tal que\ [ \ sum_ {j=1} ^p L (i_j)\ ge\ delta \]

    para cada conjunto finito\(\{I_1,I_2, \dots, I_p\}\) de intervalos abiertos que cubren\(E_\rho\). Si\(P= \{x_0,x_1, \dots,x_n\}\) es una partición de\([a,b]\), entonces\ [\ begin {ecuación}\ label {eq:3.5.4} S (P) -s (P) =\ suma_ {j\ in A} (m_j-m_j) (x_j-x_ {j-1}) +\ sum_ {j\ in B} (m_j-m_j) (x_j-x_ {j-1}), \ end {ecuación}\] donde\ [ A=\ set {j} {[x_ {j-1}, x_j]\ cap E_\ rho\ ne\ emptyset}\ mbox {\ quad y\ quad} B=\ set {j} {[x_ {j-1}, x_j] \ cap E_\ rho=\ emptyset}\ negthickspace. \]

    Dado que\(\bigcup_{j\in A} (x_{j-1},x_j)\) contiene todos los puntos de\(E_\rho\) excepto cualquiera de\(x_0\)\(x_1\),,,\(x_n\) que pueda estar en\(E_\rho\), y cada una de estas finitamente muchas posibles excepciones pueden ser cubiertas por un intervalo abierto de longitud tan pequeño como nos plazca, nuestra suposición\(E_\rho\) implica que\ [ \ sum_ {j\ en A} (x_j-x_ {j-1})\ ge\ delta. \] Además, si\(j\in A\), entonces\ [ m_j-m_j\ ge\ rho, \] así implica que\ [ S (P) -s (P)\ ge\ rho\ suma_ {j\ in A} (x_j-x_ {j-1})\ ge\ rho\ delta. \] Dado que esto se mantiene para cada partición de\([a,b]\), no\(f\) es integrable en\([a,b]\), por Teorem~. Esto demuestra que la condición declarada es necesaria para la integrabilidad.

    Por suficiencia, dejar\(\rho\) y\(\delta\) ser números positivos y dejar\(I_1\),,\(I_2\),\(I_p\) ser intervalos abiertos que cubran\(E_\rho\) y satisfagan. Dejar\ [ \ Widetilde {I} _j= [a, b]\ cap\ overline {I} _j. \] (\(\overline{I}_j=\mbox{closure of } I\).) Después de combinar cualquiera de\(\widetilde{I}_1\),\(\widetilde{I}_2\),\(\widetilde{I}_p\) que se superponen, obtenemos un conjunto de subintervalos cerrados disjuntos por pares\ [ C_j= [\ alpha_j,\ beta_j],\ quad 1\ le j\ le q\ (\ le p),\] de\([a,b]\) tal manera que \ [\ begin {ecuación}\ label {eq:3.5.5} a\\ le alpha_1<\ beta_1<\ alpha_1<\ alpha_1<\ alpha_2<\ beta_2\ cdots< \ alpha_ {q-1} <\ beta_ {q-1} <\ alpha_q<\ beta_q\ le b, \ end {ecuación}\]\ [\ begin {ecuación}\ label {eq:3.5.6} \ suma_ {i=1} ^q\, (\ beta_i-\ alpha_i) <\ delta \ end {ecuación}\] y\ [ w_f (x) <\ rho,\ quad\ beta_j\ le x\ le\ alpha_ {j+1},\ quad 1\ le j\ le q-1. \] También,\(w_f(x)<\rho\) para\(a\le x\le\alpha_1\) si\(a<\alpha_1\) y para\(\beta_q\le x\le b\) si\(\beta_q<b\).

    Dejar\(P_0\) ser la partición de\([a,b]\) con los puntos de partición indicados en, y refinar\(P_0\) particionando cada subintervalo\([\beta_j,\alpha_{j+1}]\) (así como\([a,\alpha_1]\) si\(a<\alpha_1\) y\([\beta_q,b]\) si\(\beta_q<b\)) en subintervalos en los que la oscilación de no\(f\) es mayor que \(\rho\). Esto es posible por Lemma~. De esta manera, después de renombrar toda la colección de puntos de partición, obtenemos una partición\(P=\{x_0,x_1, \dots,x_n\}\) de\([a,b]\) para la cual se\(S(P)-s(P)\) puede escribir como en, con \ [\ sum_ {j\ in A}\, (x_j-x_ {j-1}) =\ sum_ {i=1} ^q\, (\ beta_i-\ alpha_i) < \ delta\] (ver) y\ [ m_j-m_j\ le\ ho,\ quad j\ en B. \] Para esta partición,\ [ \ sum_ {j\ in A}\, (m_j-m_j) (x_j-x_ {j-1})\ Le2k\ suma_ {j\ in A}\, (x_j-x_ {j-1}) <2K\ delta, \] donde\(K\) es un límite superior para\(|f|\) on\([a,b]\) y\ [ \ sum_ {j\ in B}\, (m_j-m_j) (x_j-x_ {j-1})\ le\ rho (b-a). \] Ahora hemos demostrado que si\(\rho\) y\(\delta\) son números positivos arbitrarios, hay una partición\(P\) de\([a,b]\) tal manera que\ [\ begin {ecuación}\ label {eq:3.5.7} S (P) -s (P) <2K\ delta+\ rho (b-a). \ end {ecuación}\] Si\(\epsilon>0\), vamos\ [ \ delta=\ frac {\ épsilon} {4K}\ mbox {\ quad y\ quad}\ rho=\ frac {\ épsilon} { 2 (b-a)}. \] Entonces rinde\ [ S (P) -s (P) <\ épsilon, \] y Teorem~ implica que\(f\) es integrable en\([a,b]\).

    Necesitamos la siguiente definición para establecer la condición de integrabilidad de Lebesgue.

    -3em

    Tenga en cuenta que cualquier subconjunto de un conjunto de Lebesgue medida cero también es de Lebesgue medida cero. (¿Por qué?)

    -2em

    También hay conjuntos no denumerables de Lebesgue medida cero, pero está más allá del alcance de este libro discutir ejemplos.

    El siguiente teorema es el principal resultado de esta sección.

    Del Teorem~,\ [ S=\ set {x\ in [a, b]} {w_f (x) >0}\ negthickspace. \] Ya que\(w_f(x)>0\) si y solo si\(w_f(x)\ge1/i\) para algún entero positivo\(i\), podemos escribir\ [\ begin {ecuación}\ label {eq:3.5.12} S=\ bigcup^\ infty_ {i=1} s_i, \ end {ecuación}\] donde\ [ s_i=\ set {x\ in [a, b]} {w_f (x)\ ge1/i}. \]

    Ahora supongamos que\(f\) es integrable en\([a,b]\) y\(\epsilon>0\). De Lemma~, cada uno\(S_i\) puede ser cubierto por un número finito de intervalos abiertos\(I_{i1}\),\(I_{i2}\),,\(I_{in}\) de longitud total inferior a\(\epsilon/2^i\). Simplemente renumeramos estos intervalos consecutivamente; así,\ [ I_1, I_2,\ dots= I_ {11},\ dots, I_ {1n_1}, I_ {21},\ dots, I_ {2n_2},\ dots, I_ {i1},\ dots, I_ {in_i},\ dots. \] Ahora y sostenga por y, y hemos demostrado que la condición declarada es necesaria para la integrabilidad.

    Para la suficiencia, supongamos que la condición declarada se mantiene y\(\epsilon>0\). Entonces\(S\) puede ser cubierto por intervalos abiertos\(I_1,I_2, \dots\) que satisfagan. Si\(\rho>0\), entonces el conjunto\ [ E_\ rho=\ set {x\ in [a, b]} {w_f (x)\ ge\ rho} \] de Lemma~ está contenido en\(S\) (Teorem~), y por lo tanto\(E_\rho\) está cubierto por\(I_1,I_2, \dots\). Dado que\(E_\rho\) está cerrado (Lemma~) y acotado, el teorema de Heine—Borel implica que\(E_\rho\) está cubierto por un número finito de intervalos desde\(I_1,I_2, \dots\). La suma de las longitudes de este último es menor que\(\epsilon\), por lo que Lemma~ implica que\(f\) es integrable en\([a,b]\).


    This page titled 3.1: Definición de la Integral is shared under a CC BY-NC-SA 3.0 license and was authored, remixed, and/or curated by William F. Trench.